LSAT and Law School Admissions Forum

Get expert LSAT preparation and law school admissions advice from PowerScore Test Preparation.

 Francis O'Rourke
PowerScore Staff
  • PowerScore Staff
  • Posts: 471
  • Joined: Mar 10, 2017
|
#45784
Hi Hskhader!

It's a good idea to investigate a speaker's usage of "many." This stimulus however does not make the flaw you are seeing.

While it is true, as you point out, that some herbs may still be unsafe for consumers, the speaker's conclusion did not make a claim about the safety of all herbs. Rather, the speaker claimed that advocates of "these herbs" ought to be allowed to prescribe them. The word "these" tells us that the conclusion does not concern herbs that may still pose risks.

I hope this helps! Let us know if you have any further questions :-D
 LSAT2018
  • Posts: 242
  • Joined: Jan 10, 2018
|
#57991
David Boyle wrote:
Not always. --Law Services doesn't like to be sued, one hears, so that just in case they pick two answers that both weaken, they sometimes say "most weaken" so that they can't be sued by some test taker who screams "Hey! There's more than one right answer!"
Just see which answer most weakens, that's all. If one answer could maybe weaken a tiny little bit, and another would weaken hugely, pick the second one.

As for question 5: answer A is good because even if herbal remedies are harmless, answer A shows that those remedies *displace* better, conventional remedies. Obviously, this could hurt the patient.

Hope this helps,
David

Although I understand why answer (A) is correct, that it MOST weakens the argument. But I couldn't help but feel that the other answers may weaken the argument as well. Answer (B) for instance may weaken because if the herbal remedies are marketed with claims of proven effectiveness, this could mislead the patients. This applies to answer (D) as well as the profit-oriented actions of the purveyors could mislead the patients.

What can distinguish these answers from one another? When eliminating the answers, I looked at the strength of the answers based on 'many practitioners' in answer (A), 'many herbal remedies' in answer (B), and 'some patients' in answer (C). These seemed to be very weak compared to answers (D) and answers (E). Sometimes, for weaken questions, it becomes important to compare the strength of these answers, but in this case, it was not as important. What am I missing here?
 Jay Donnell
PowerScore Staff
  • PowerScore Staff
  • Posts: 144
  • Joined: Jan 09, 2019
|
#63194
Hi LSAT2018!

I think the major issue here that is muddling up the answers more than we'd like is your focus on the answer choice strength more so than the exact phrasing of the conclusion we are here to attack.

The author concludes that "advocates of these herbs as remedies for serious illnesses should always be allowed to prescribe them."

That always is awfully strong, and that is exactly where the correct answer most successfully weakens the argument.

You're right that the strength of an answer can very often be relevant, but that often falls secondary to the exact terminology used in the answers and the conclusion in the stimulus.

Here, that 'always' in the conclusion is so strong that it wouldn't take a large amount of power to dispute that claim with a counterexample or objection, and even the 'many' in A is more than enough to bring serious doubt as to whether these herbs should always be prescribed, as for at least those 'many,' the patients would use the herbs at the expense of more effective remedies.

Hope that helps!
User avatar
 PresidentLSAT
  • Posts: 87
  • Joined: Apr 19, 2021
|
#100687
I hope I get a response based on my reasoning.

I would have picked A if it read, "many practitioners and patients WILL neglect more effective conventional medicines in favor of herbal remedies."

The difference here (for me) is that if we allow what the doctor is advocating for, we are setting ourselves up for a phenomenon that won't help at all.

The way the answer was framed suggests it's an already existing phenomenon, and if that's the case, I'm sure the doctor is already aware of it before advocating for her claim.

Or I'm reading into it too much lol?
 Robert Carroll
PowerScore Staff
  • PowerScore Staff
  • Posts: 1819
  • Joined: Dec 06, 2013
|
#100721
PresidentLSAT,

The tense doesn't make a difference. Further, from where are you logically inferring that the doctor already knows about the phenomenon if it's happening? This seems to violate a fundamental LSAT skill - don't assume anything unless it's stated. There is no common sense reason to think the doctor already knows about what answer choice (A) says. Otherwise, every Weaken answer choice would fail - we could assume the author already knew about each such choice and therefore that choice would not be available to weaken the argument. In fact...if answer choice (A) said "will" instead, couldn't the same objection be lodged against it? The doctor could be assumed to know what will happen!

Robert Carroll

Get the most out of your LSAT Prep Plus subscription.

Analyze and track your performance with our Testing and Analytics Package.